LSAT Logical Reasoning Question Types

Réussis tes devoirs et examens dès maintenant avec Quizwiz!

The dialogue provides the most support for the claim that Sanchez and Merriwheather disagree over whether the school.

Point at Issue. Use the Kaplan Decision Tree.

Bell and Soltan disagree with each other about which one of the following?

Point at Issue. Use the Kaplan Point at Issue Tree.

Which one of the following is an assumption on which the argument depends?

Necessary assumption. The question directly asks for an assumption, and one on which the argument DEPENDS, making this a Necessary Assumption question.

Which of the following, if true, most helps to resolve the apparent conflict in the statement above?

Paradox. Question asks for something that will "resolve the apparent conflict." That means there will be a mystery to be solved.

Which of the following, if true, most helps to resolve the apparent discrepancy in the information above?

Paradox. Question asks for something that would "resolve the apparent discrepancy" making this a Paradox question.

Which of the following, if true, most helps to explain the failure of the strategy involving wolves?

Paradox. The question asks for something that "helps to explain" a situation. If a situation needs explaining, that indicates a paradox question.

Which one of the following, if true, most helps to resolve the apparent discrepancy described above.

Paradox. The question asks for something that will "resolve the apparent discrepancy" making this a Paradox Q.

Which one of the following, if true, would most help to resolve the puzzle described above?

Paradox. The question asks for something that will "resolve the puzzle described" making this a Paradox.

Which one of the following arguments exhibits a flawed pattern of reasoning most similar to that exhibited by the argument above?

Parallel Flaw. The correct answer will be an argument "most similar to" the one in the stimulus, and that reasoning is described as FLAWED/ That makes this a Parallel Flaw question.

The pattern of reasoning in the argument above is most similar to that in which one of the following arguments?

Parallel Reasoning. Correct answer will use the exact same logical structure.

Which one of the following arguments is most similar in its reasoning to the employee's argument?

Parallel Reasoning. The correct answer will be an argument "most similar" to the one presented in the stimulus.

The flawed reasoning exhibited by the argument above is most similar to that exhibited by which one of the following?

Parallel flaw. The correct answer will be an argument with reasoning "most similar to" that in the given argument. That reasoning is said to be flawed, so this is a Parallel Flaw question.

The dialogue provides the most support for inferring that Liang and Sarah agree with each other that

Point at Issue (agree). Use Kaplan Point at Issue Tree.

The dialogue provides the most support for the claim that Peraski and Jackson disagree over whether

Point at Issue.

Which one of the following is an assumption that the oceanographer's argument requires?

Necessary Assumption. The question directly asks for an assumption, and one that the argument REQUIRES.

Which of the following is an assumption required by the argument?

Because the assumption is required, it is a Necessary Assumption question.

Which of the following is an assumption on which the argument depends?

Necessary Assumption.

The reasoning in the argument is flawed because the argument

Flaw.

The reasoning in the argument is flawed in that the argument

Flaw.

Which one of the following most accurately describes a flaw in the airport administrator's argument?

Flaw.

The reasoning in the argument is most vulnerable to criticism on the grounds that the argument

Flaw. "vulnerable to criticism" indicates a Flaw.

The spokesperson's argument is most vulnerable to criticism on the grounds that it

Flaw. The correct answer will describe why the argument is "vulnerable to criticism."

The reasoning in the community organizer's argument is most vulnerable to criticism on the grounds that the argument

Flaw. The correct answer will describe why the given argument is "vulnerable to criticism" which is common wording for a Flaw Q.

Of the following claims, which one can most justifiably be rejected on the basis of the statement above?

Inference. Ground for choosing teh correct answer will be made "on the basis of the statement above." This makes it an inference. However, the correct answer will not be true or supported as valid, the correct answer will be REJECTED, which means the stimulus will be used to contradict the correct answers. Answer choices that could be true or must be true can be rejected.

if the critic's statements are true, which one of the following must also be true?

Inference. The correct answer "must be true" based on the information given, making this an inference question.

Which of the following is most strongly supported by the information above?

Inference. The correct answer will be "strongly supported by the information" given, making this an inference question.

If the statements above are true, which one of the following must also be true?

Inference. The question asks for something that "must be true" based on the given information. That makes this an inference question.

Which one of the following can most reasonably be concluded on the basis of the information above?

Inference. The question asks for something that can "reasonably be concluded" from the information given. That conclusion will not be directly stated, but it will be an inference based on what's provided.

The statements above, if true, most strongly support which one of the following?

Inference. The stimulus will "strongly support" the correct answer, meaning the correct answer will be a logical inference.

which of the following most accurately expresses the overall conclusion drawn in the argument?

Main Point

Which one of the following most accurately expresses the main conclusion of the pundit's argument?

Main Point.

Which of the following most accurately expresses the conclusion drawn in the citizen's argument?

Main Point. "conclusion drawn" = MP.

Garza responds to Patterson by doing which of the following?

Method of Argument. The phrase "by doing which of the following" indicates the question is asking HOW Garza responds as opposed to WHAT Garza says, making this a Method of Argument Q.

The principle stated above, if valid, most helps to justify the reasoning in which one of the following arguments?

Principle (Apply/Inference). The correct answer will be a specific argument that is justified by a principle given in the stimulus. That makes this an Apply the Principle question.

The engineer's argument can most reasonably be interpreted as invoking which one of the following principles?

Principle (Identify/Inference). The question asks directly for a principle that is used by the engineer. The correct answer will be a general rule that matches logically to the specific argument made by the engineer. That makes this an Identify the Principle question.

Which of the following principles, if valid, most helps to justify the columnist's judgment that the legislator's proposal is unacceptable?

Principle (Identify/Strengthen). Because the principle will be listed in the answer choices, this is an Identify the Principle q. Further, because the correct answer will help "justify the columnist's judgment," it will also act like a strengthen question.

Which one of the following principles, if valid, most helps to justify the politician's criticism?

Principle (Identify/Strengthen). The correct answer will be a principle that will be used to JUSTIFY a specific argument in the stimulus. That makes this an Identify the Principle question that acts like a Strengthen question.

Which of the following, if true, most strengthens the argument?

Strengthen. Asks for something that strengthens the given argument.

Which one of the following, if true, adds the most support for the conclusion of the argument?

Strengthen. Question asks for something that "adds the most support" for the argument.

Which one of the following, if true, provides the most support for the argument in the editorial?

Strengthen. The question asks for something that "provides the most support" for the given argument. That means it will strengthen that argument.

Which of the following, if true, most strengthens the support for the ecologist's conclusion?

Strengthen. The question directly asks for something that will strengthen the given argument.

The answer to which one of the following questions would most help in evaluating the argument?

Strengthen/Weaken (Evaluate the argument). The question asks for something that would "help in evaluating the argument." That makes this an Evaluate the Argument variant of Strengthen/Weaken. The correct answer will pose a question that, depending on how it's answered, would validate or invalidate the argument.

The conclusion drawn above follows logically if which one of the following is assumed to be true at Tromen University?

Sufficient Assumption. The question asks for something that, IF assumed, would allow the conclusion to be drawn.

The conclusion of the argument follows logically if which one of the following is assumed?

Sufficient Assumption. The question asks for something that, IF assumed, would logically complete the argument.

Which one of the following is a statement from which the conclusion can be properly drawn using the principle?

Sufficient Assumption. The stimulus contains a conclusion and the correct answer will provide an unstated piece of information that, when added to the given evidence, will allow the conclusion to be properly drawn.

The conclusion of the argument can be properly drawn if which one of the following is assumed?

Sufficient Assumption. the "if" makes this a sufficient assumption.

Which of the following, if true, most strengthens the farmer's argument?

The question directly asks for something that will strengthen the farmer's argument.

Which of the following, if true, casts the most doubt on the politician's argument?

Weaken. QUestion asks for something that "casts the most doubt" on the given argument, making it a weakening question.


Ensembles d'études connexes

HESI Patient Review--Pediatrics-Out-patient Peds (Adolescents)

View Set

Chapter 7: Anatomy and Physiology of Pregnancy (includes chp 12 Q) NCLEX 59Qw/exp ***Definitely LOOK OVER ***

View Set

Biology 1107: Chapter 7 & 8 Homework

View Set

A nurse is caring for a client who reports pain. When documenting the quality of the clients pain on an initial pain assessment. the nurse should record which of the fallowing client statements?

View Set

Biology Final Study Guide: Chapter 48

View Set

(PrepU) Chapter 14: Assessing Skin, Hair, and Nails

View Set